K
Khách

Hãy nhập câu hỏi của bạn vào đây, nếu là tài khoản VIP, bạn sẽ được ưu tiên trả lời.

5 tháng 3 2020

1

\(a\sqrt{b^3+1}=a\sqrt{\left(b+1\right)\left(b^2-b+1\right)}\le a\cdot\frac{b+1+b^2-b+1}{2}=\frac{ab^2}{2}+1\)

Tương tự ta có:\(P\le3+\frac{1}{2}\left(ab^2+bc^2+ca^2\right)\)

Giả sử b nằm giữa a và c

Ta có:

\(\left(b-a\right)\left(b-c\right)\le0\Leftrightarrow b^2-bc-ab+ac\le0\Leftrightarrow b^2+ac\le ab+bc\)

\(\Leftrightarrow ab^2+a^2c\le a^2b+abc\Leftrightarrow ab^2+bc^2+ca^2\le a^2b+bc^2+abc\)

\(\le a^2b+bc^2+2abc=b\left(a+c\right)^2=b\left(3-b\right)^2\)

Ta chứng minh \(b\left(3-b\right)^2\le4\) dể chứng minh

Khi đó:\(P\le3+\frac{4}{2}=5\)

Dấu "=" xảy ra tại a=0;b=1;c=2 và các hoán vị

2

Đặt \(a+b-c=x;b+c-a=y;c+a-b=z\)

\(\Rightarrow a=\frac{x+y}{2};b=\frac{y+z}{2};c=\frac{z+x}{2}\)

Bất đẳng thức cần chứng minh tương đương với:\(xyz\le\frac{\left(x+y\right)\left(y+z\right)\left(z+x\right)}{8}\) ( đúng theo bđt cô si )

P/S:a,b,c không là độ dài 3 cạnh tam giác vẫn đúng theo BĐT Schur

5 tháng 3 2020

Bài 1: em làm không đúng rồi và cô không hiểu ý tưởng làm bài của em nhưng có mấy lỗi cơ bản: 

Sai dòng thứ nhất \(\frac{ab^2}{2}+a\)

Dấu bằng xảy ra cũng sai. Dòng thứ 6 em nhân cả hai vế cho a mà dấu bằng a = 0 . vô lí

Dòng thứ 5 ( b - a ) ( b  - c ) <= 0 thì dấu bằng xảy ra a = b hoặc b = c chứ 

Dòng thứ 8 => sau đó làm thế nào. 

24 tháng 5 2018

Áp dụng BĐT Cauchy-Schwarz và Nesbitt ta có:

\(P\le\sqrt{\left(1+1+1\right)\left(3-\left(\frac{a}{b+c}+\frac{b}{a+c}+\frac{c}{a+b}\right)\right)}\)

\(\le\sqrt{\left(1+1+1\right)\left(3-\frac{3}{2}\right)}=\frac{3\sqrt{2}}{2}\)

25 tháng 5 2018

rõ đi bạn mình không hiểu lắm

31 tháng 8 2021

Ta có: \(abc=b+2c\)

\(\Rightarrow a=\dfrac{b+2c}{bc}\)\(\Rightarrow a=\dfrac{1}{c}+\dfrac{2}{b}\)

Áp dụng bất đẳng thức: \(\dfrac{1}{a}+\dfrac{1}{b}\ge\dfrac{4}{a+b}\)

Ta có: \(\dfrac{3}{b+c-a}+\dfrac{4}{c+a-b}+\dfrac{5}{a+b-c}\)

\(=\dfrac{1}{b+c-a}+\dfrac{1}{c+a-b}+2\left(\dfrac{1}{b+c-a}+\dfrac{1}{a+b-c}\right)+3\left(\dfrac{1}{c+a-b}+\dfrac{1}{a+b-c}\right)\ge\dfrac{4}{b+c-a+c+a-b}+2.\dfrac{4}{b+c-a+a+b-c}+3.\dfrac{4}{c+a-b+a+b-c}=\dfrac{4}{2c}+2.\dfrac{4}{2b}+3.\dfrac{4}{2a}=\dfrac{2}{c}+\dfrac{4}{b}+\dfrac{6}{a}=2\left(\dfrac{1}{c}+\dfrac{2}{b}+\dfrac{3}{a}\right)=2\left(a+\dfrac{3}{a}\right)\ge2.2\sqrt{\dfrac{a.3}{a}}=4\sqrt{3}\)

(bất đẳng thức Cauchy cho 2 số dương)

\(ĐTXR\Leftrightarrow a=b=c=\sqrt{3}\)

 

AH
Akai Haruma
Giáo viên
13 tháng 3 2022

1. Đặt $\frac{a}{b+c}+\frac{b}{c+a}+\frac{c}{a+b}=T$

$\frac{a}{b+c}> \frac{a}{a+b+c}$
$\frac{b}{c+a}> \frac{b}{c+a+b}$

$\frac{c}{a+b}> \frac{c}{a+b+c}$
$\Rightarrow T> \frac{a+b+c}{a+b+c}=1$ (đpcm) 

----

Xét hiệu:

$\frac{a}{b+c}-\frac{2a}{a+b+c}=\frac{-a(b+c-a)}{(b+c)(a+b+c)}<0$ theo BĐT tam giác

$\Rightarrow \frac{a}{b+c}< \frac{2a}{a+b+c}$ 

Tương tư: $\frac{b}{c+a}< \frac{2b}{c+a+b}$

$\frac{c}{a+b}< \frac{2c}{a+b+c}$

Cộng theo vế:

$T< \frac{2(a+b+c)}{a+b+c}=2$

 

$\frac{b}{a+c}

AH
Akai Haruma
Giáo viên
13 tháng 3 2022

2. 

Áp dụng BĐT AM-GM:

\(\frac{b+c}{a}.1\leq \frac{1}{4}(\frac{b+c}{a}+1)^2=\frac{(b+c+a)^2}{4a^2}\)

\(\Rightarrow \sqrt{\frac{a}{b+c}}\geq \frac{2a}{a+b+c}\)

Tương tự với các phân thức còn lại và cộng theo vế:
$\Rightarrow T\geq \frac{2(a+b+c)}{a+b+c}=2$

Dấu "=" xảy ra khi $b+c=a; c+a=b; a+b=c\Rightarrow a=b=c=0$ (vô lý)

Vậy dấu "=" không xảy ra, tức là $T>2>1$ (đpcm)

 

20 tháng 5 2018

Do a, b, c là 3 cạnh của tam giác ABC nên a, b, c đều dương. Do đó cả 2 vế đều dương.

Lập phương mỗi vế, ta được phương trình mới tương đương với phương trình đã cho:

\(\frac{a^3}{b^3+c^3}+\frac{b^3}{c^3+a^3}+\frac{c^3}{a^3+b^3}< 8\cdot4=32\left(1\right)\)

Ta có \(\frac{a^3}{b^3+c^3}< \frac{2a^3}{a^3+b^3+c^3}\);\(\frac{b^3}{a^3+c^3}< \frac{2b^3}{a^3+b^3+c^3}\)và \(\frac{c^3}{a^3+b^3}< \frac{2c^3}{a^3+b^3+c^3}\)

Do đó \(\frac{a^3}{b^3+c^3}+\frac{b^3}{c^3+a^3}+\frac{c^3}{a^3+b^3}< 2< 32\)

Vì vậy bất đẳng thức (1) là đúng, nên bất đẳng thức đã cho là đúng